what is x if f (x) = 4x + 13 = 9

Answers

Answer 1

Answer:

-1

Step-by-step explanation:

4x + 13 = 9

4x + 13 - 13 = 9 -13

4x = -4

-4/4 = -1

X = -1

Answer 2

Answer:

-1.

Step-by-step explanation:

4x + 13 = 9

4x =  9 -13

4x = -4

x = -4/4

X = -1


Related Questions

The equation of a straight line that is parallel to a straight line. 2y =3x-1​

Answers

The equation of the line that is parallel to 2y = 3x - 1 and passes through the point (4, 2) is: y = (3/2)x - 4

To find the equation of a straight line that is parallel to the line 2y = 3x - 1, we need to remember that parallel lines have the same slope.

First, let's rearrange the given equation into slope-intercept form, y = mx + b, where m is the slope and b is the y-intercept:

2y = 3x - 1

y = (3/2)x - 1/2

So the slope of this line is 3/2.

Now, if we want to find the equation of a line that is parallel to this line, we just need to use the same slope. Let's call the new line y = mx + b, where m is the slope we just found and b is the y-intercept we need to find.

So the equation of the parallel line is:

y = (3/2)x + b

To find the value of b, we need to use a point on the line. Let's say we want the line to go through the point (4, 2):

2 = (3/2)(4) + b

2 = 6 + b

b = -4

So the equation of the line that is parallel to 2y = 3x - 1 and passes through the point (4, 2) is: y = (3/2)x - 4

To know more about   straight line, here

https://brainly.com/question/25969846

#SPJ4

Which of the equations below could be used as a line of best fit to approximate the data in the scatterplot?
Hint: Use the Desmos Graphing Calculator to graph the table and replicate the scatter plot. Then see which line from the choices below looks the best.

Answers

The equation of the line of best fit is y = 0.883x + 17.95.

We have,

To find the line of best fit, we want to find the equation of the line that comes closest to passing through all the points in the scatterplot.


One way to do this is to use linear regression analysis.

Using a calculator or statistical software,

We can find that the equation of the line of best fit for this data is:

y = 0.883x + 17.95

Thus,

The equation of the line of best fit is y = 0.883x + 17.95.

Learn mroe about scatterplots here:

https://brainly.com/question/30017616

#SPJ1

There are 5 quadratics below. Four of them have two distinct roots each. The other has only one distinct root; find the value of that root.a. 4x^2 + 16x − 9b. 2x^2 + 80x + 400c. x^2 − 6x − 9d. 4x^2 − 12x + 9e. −x^2 + 14x + 49

Answers

Answer:

x = 3/2 or 1.5

Step-by-step explanation:

All 5 of the quadratics are in standard form, whose general form is

[tex]ax^2+bx+c[/tex]

One of the ways in which we solve quadratic equations is through the quadratic formula which is

[tex]x=\frac{-b+/-\sqrt{b^2-4ac} }{2a}[/tex], where x is the root(s)

We can find the total number of solutions a quadratic equation has using the discriminant from the quadratic formula which is

[tex]b^2-4ac[/tex]

When the discriminant is greater than 0, there is 2 distinct rootsWhen the discriminant is equal to 0, there is 1 distinct rootWhen the discriminant is less than 0, there are 0 distinct/"real" roots

(a.) For 4x^2 + 16x - 9b, 4 is our a value, 16 is our b value and -9 is our c value:

[tex]16^2-4(4)(-9)\\256+144\\400 > 0[/tex]

(b.) For 2x^2 + 80x + 400, 2 is our a value, 80 is our b value, and 400 is our c value:

[tex]80^2-4(2)(400)\\6400-3200\\3200 > 0[/tex]

(c.) For x^2 - 6x - 9, 1 is our a value, -6 is our b value and -9 is our c value

Quick fact:  for x^2 or -x^2, there's a 1 or -1 in front of the variable, but it's usually not written because it's a well known mathematical effect and it's assumed we already know this)

[tex](-6)^2-4(1)(-9)\\36+36\\72 > 0[/tex]

(d.) For 4x^2 - 12x + 9, 4 is our a value, -12 is our b value, and 9 is our c value:

[tex](-12)^2-4(4)(9)\\144-144\\0=0[/tex]

We don't have to do (e.) because we see that since the discriminant for (d.) equals 0, this is the quadratic with only one distinct solution/rootWe can now solve for this root using the quadratic formula

[tex]x=\frac{-(-12)+/-\sqrt{(-12)^2-4(4)(9)} }{2(4)}\\ \\x=\frac{12+/-\sqrt{0} }{8} \\\\x=12/8=3/2\\or\\x=1.5[/tex]

A random variable X has possible values of 1-6. Would the following value of X be included if we want at most 4? Choose yes if the value is included.

Answers

No, the value would not be included if we want at most 4. In statistics, a variable is a characteristic or attribute that can be measured or observed.

A random variable is a variable whose value is determined by chance or probability. The possible values of a random variable are called its values. In this case, the random variable X has possible values of 1-6. If we want at most 4, this means we want all the values of X that are less than or equal to 4. Therefore, the value in question (which we don't know) would only be included if it is less than or equal to 4. If it is greater than 4, then it would not be included. To summarize, whether the value of X is included or not depends on whether it is less than or equal to 4, which is the condition we have set.

Learn more about variable here:

brainly.com/question/29691945

#SPJ11

PLEASE HELP ME! THANK YOU!

Answers

Answer:

138°

235°

240°

Step-by-step explanation:

The first situation shows a straight angle, so the smaller angles that make it up should be supplementary. That means that their measures add up to 180°. My work for this first situation is shown below:

n + 42° = 180°
n + 42° - 42° = 180° - 42°
n = 138°

Now, let's look at the second situation. This one's pretty simple. It's just asking us to add up the measures of all the angles are given, so all we have to do is some addition, shown below:

23° + 40° + 92° + 80° = 235°

Finally, we're at the last situation. This situation shows a full angle, so the smaller angles that make it up should add up to 360°, since this is the amount of degrees in a full circle. Again, if we know the measure of one of these angles, we should be able to find the measure of the other one. See my work below:

n + 120° = 360°
n + 120° - 120° = 360° - 120°
n = 240°

And there are all your answers! Let me know if you need further clarification on all that. :)


A density graph for all of the possible temperatures from 60 degrees to 160
degrees can be used to find which of the following?
OA. The probability of a temperature from 90 degrees to 180 degrees
OB. The probability of a temperature from 30 degrees to 120 degrees
OC. The probability of a temperature from 30 degrees to 90 degrees
OD. The probability of a temperature from 90 degrees to 120 degrees

Answers

Answer:

The density graph for all possible temperatures from 60 degrees to 160 degrees can be used to find the probability of a temperature falling within a certain range.

Option (A) is incorrect because it includes temperatures that are outside the range of the graph.

Option (B) is incorrect because it includes temperatures that are outside the range of the graph.

Option (C) is incorrect because it includes temperatures that are outside the range of the graph.

Option (D) is the only option that falls within the range of the graph. Therefore, the density graph can be used to find the probability of a temperature from 90 degrees to 120 degrees, which is option (D).

Step-by-step explanation:

The line graph shows the number of pairs of shoes owned
by some children
a)
Number of children
3
2
1
0
2 3 4 5 6
3 4
Number of pairs of shoes
0
1 2
What is the modal number
of pairs of shoes owned by the
children?
b) What is the median number
of pairs of shoes owned by the
children?
c) What is the mean number of
pairs of shoes owned by the
children?

Answers

1. The modal number of pairs of shoes owned by the children will be; 3.

2. The median number of pairs of shoes owned by the children  will be;3.

3. The Mean is 3.

1. The modal number of pairs of shoes owned by the children would be 3.

2. The median number of pairs of shoes owned by the children are;

= 14/2 th term

= 7 th term

= 3

3. The Mean would be

= (1 x 2+ 2 x 3+ 3 x 5+ 4 x 2 + 5 x 1+ 6x 1)/ (2 +3 +5 +2 + 1 +1)

= 42/14

= 3

Learn more about Arithmetic Mean here:

brainly.com/question/13000783

#SPJ1

The histogram shows data collected about the number of passengers using city bus transportation at a specific time of day.

A histogram titled City Bus Transportation. The x-axis is labeled Number Of Passengers and has intervals of 1 to 10, 11 to 20, 21 to 30, 31 to 40, and 41 to 50. The y-axis is labeled Frequency and starts at 0 with tick marks every 1 units up to 9. There is a shaded bar for 1 to 10 that stops at 3, for 11 to 20 that stops at 3, for 21 to 30 that stops at 7, for 31 to 40 that stops at 4, and for 41 to 50 that stops at 3.

Which of the following data sets best represents what is displayed in the histogram?

A: (4, 5, 7, 8, 10, 12, 13, 15, 18, 21, 23, 28, 32, 34, 36, 40, 41, 41, 42, 42)
B: (4, 7, 10, 13, 14, 19, 22, 24, 26, 27, 29, 31, 33, 35, 36, 38, 40, 42, 42, 42)
C: (4, 5, 7, 8, 12, 13, 15, 18, 19, 21, 24, 25, 26, 28, 29, 30, 32, 33, 35, 42)
D: (4, 6, 9, 12, 16, 18, 21, 24, 25, 26, 28, 29, 30, 32, 35, 36, 38, 41, 41, 42)

Answers

The best data set that represents the histogram is (4, 5, 7, 8, 12, 13, 15, 18, 19, 21, 24, 25, 26, 28, 29, 30, 32, 33, 35, 42, option C is correct.

From the histogram, we can see that there were 3 data points in the interval 1-10, 3 data points in the interval 11-20, 7 data points in the interval 21-30, 4 data points in the interval 31-40, and 3 data points in the interval 41-50.

Therefore, the best data set that represents the histogram is the one that has 3 data points in the range 1-10, 3 data points in the range 11-20, 7 data points in the range 21-30, 4 data points in the range 31-40, and 3 data points in the range 41-50.

4, 5, 7, 8, 10, 12, 13, 15, 18, 21, 23, 28, 32, 34, 36, 40, 41, 41, 42, 42 does not fit this pattern since it has more than 3 data points in some of the intervals.

4, 7, 10, 13, 14, 19, 22, 24, 26, 27, 29, 31, 33, 35, 36, 38, 40, 42, 42, 42 also does not fit the pattern since it has more than 3 data points in some of the intervals.

4, 5, 7, 8, 12, 13, 15, 18, 19, 21, 24, 25, 26, 28, 29, 30, 32, 33, 35, 42 fits the pattern and has 3 data points in each interval. This is the correct answer.

4, 6, 9, 12, 16, 18, 21, 24, 25, 26, 28, 29, 30, 32, 35, 36, 38, 41, 41, 42 does not fit the pattern since it has more than 3 data points in some of the intervals.

Therefore, the best data set that represents the histogram is  (4, 5, 7, 8, 12, 13, 15, 18, 19, 21, 24, 25, 26, 28, 29, 30, 32, 33, 35, 42

To learn more on Statistics click:

https://brainly.com/question/30218856

#SPJ1

in a certain lottery, you must choose three numbers: any number between 1 and 10; any number between 1 and 20; and any number between 1 and 30. numbers may repeat and order matters (e.g., 5-5-5 is allowed; and 5-9-30 is different than 9-5-30). how many different lottery picks are there? enter as a whole number.

Answers

Answer: 6000

Step-by-step explanation:

For the numbers you choose, there are 10, then 20, then 30 possible numbers to choose from.

To find the total amount of possible combinations with repetition, you just do 10x20x30 = 6000.

What is mzQPS?
A. 68
B. 71
C. 84
D. cannot be determined

Answers

The value of the angle m<QPS cannot be determined. Option D

How to determine the value

To determine the value of the angle, we need to take note of the properties of a parallelogram

The opposite sides are parallel and equal.The opposite angles are also equal.The consecutive or adjacent angles are supplementary, that is, they sum up to 180 degrees

From the information given, we have that;

m<R = 6x -14

m<Q = 3x + 8

m< S= 5x + 4

The value of the angle , QPS cannot be determined because the angle is not represented with any variable for calculation

Learn more about parallelogram at: https://brainly.com/question/10744696

#SPJ1

R= 6.45. Find the area of the circle shown. Use 3.14 for π . Round to the nearest hundredth if necessary.

Answers

Answer:

130.63

Step-by-step explanation:

The formula for a circle is:

Area = πr²

Let's plug in our values.

Area = 3.14(6.45)²

= 3.14(41.6025)

= 130.63185

Now we round to the nearest hundredth to get 130.63.

Hope this helps and good luck on your homework!

Answer:

130.62

Step-by-step explanation:

The formula for the area of a circle is [tex]\pi r^{2}[/tex].

We use 3.14 for pi and 6.45 for r:

3.14 · 6.45²

3.14 · 41.6

130.624, rounded to the nearest hundredth 130.62

The variance and standard deviation can never be
zero
negative
smaller than the mean
larger than the mean

Answers

The variance and standard deviation can never be negative. However, they can be zero if there is no variability in the data. It is possible for the variance and standard deviation to be smaller or larger than the mean depending on the spread of the data.

The variance and standard deviation can never be negative.
1. Variance is a measure of how spread out the data points are from the mean. It is calculated by finding the average of the squared differences from the mean. Since squares are always positive or zero, the variance cannot be negative.

2. Standard deviation is the square root of the variance. Since the square root of a negative number is not a real number, the standard deviation cannot be negative either.

It is worth noting that both variance and standard deviation can be zero if all data points are the same, and they can be smaller or larger than the mean, depending on the data distribution.

Learn more about Variance:

brainly.com/question/13708253

#SPJ11

What are all the different ways to choose the ones digit, A, in the number 572435 A, so that the number will be divisible by 3? Explain your reasoning.

Answers

The different ways to choose the ones digit A are:

- A can be any digit if the sum of the first 6 digits is divisible by 3.
- A can be 2, 5, or 8 if the sum of the first 6 digits leaves a remainder of 1 when divided by 3.
- A can be 1, 4, 7, or 0 if the sum of the first 6 digits leaves a remainder of 2 when divided by 3.

To determine if a number is divisible by 3, we can add up the digits and check if the sum is divisible by 3. For the number 572435A, the sum of the first 6 digits is 5 + 7 + 2 + 4 + 3 + 5 = 26. We need to add a number A to this sum to make it divisible by 3.

There are three possible scenarios to make the sum divisible by 3:

1. If the sum of the first 6 digits is already divisible by 3, then any digit A can be added to the end to make the number divisible by 3. In this case, the sum of the first 7 digits will also be divisible by 3.

2. If the sum of the first 6 digits leaves a remainder of 1 when divided by 3, then A must be 2, 5, or 8. Adding any other digit to the end will result in a sum that is not divisible by 3. In this case, adding 2, 5, or 8 to the end of the number will result in a sum of digits that is divisible by 3.

3. If the sum of the first 6 digits leaves a remainder of 2 when divided by 3, then A must be 1, 4, 7, or 0. Adding any other digit to the end will result in a sum that is not divisible by 3. In this case, adding 1, 4, 7, or 0 to the end of the number will result in a sum of digits that is divisible by 3.

Therefore, the different ways to choose the ones digit A in the number 572435A so that the number will be divisible by 3 are:

- A can be any digit if the sum of the first 6 digits is divisible by 3.
- A can be 2, 5, or 8 if the sum of the first 6 digits leaves a remainder of 1 when divided by 3.
- A can be 1, 4, 7, or 0 if the sum of the first 6 digits leaves a remainder of 2 when divided by 3.

To learn more about permutations visit : https://brainly.com/question/1216161

#SPJ11

(b) If the critical value is 4.605 at a significance level of 0.10, can we reject the null hypothesis? State your reason. (3 marks) QUESTION A6 (5 marks) An article studied the relation between the number of accidents, y, and the difference between the width of the bridge and roadway, x, (in feet) in a city. The author had developed its regression equation, y= 74.7 - 6.44x.
(a) State the dependent and independent variables for the above problem. (2 marks) (b) Estimate the number of accidents occurred if the difference of the width is 8 feet. (3 marks)

Answers

(a) The dependent variable is the number of accidents, y. The independent variable is the difference between the width of the bridge and roadway, x.

(b) To estimate the number of accidents if the difference of the width is 8 feet, we substitute x = 8 into the regression equation:

y = 74.7 - 6.44(8) = 24.58

Therefore, we estimate that there would be 24.58 accidents if the difference of the width is 8 feet.

As for the earlier question, the answer would be:

We need to calculate the test statistic to determine if we can reject the null hypothesis. The test statistic is calculated as:

test statistic = (sample mean - null hypothesis value) / (standard error of the sample mean)

Since the question does not provide any sample mean or standard error, we cannot calculate the test statistic. Therefore, we cannot determine if we can reject the null hypothesis based on the critical value alone.

https://brainly.com/question/31700335

#SPJ11

Which expression is equivalent to 4−3(x+2)+2(3−2x)?

Answers

The expression is equivalent to 4 − 3(x + 2) + 2(3 − 2x) will be 4 − 7x. Then the correct option is A.

Given that:

Expression, 4 − 3(x + 2) + 2(3 − 2x)

The equivalent is the expression that is in different forms but is equal to the same value.

Simplify the expression, then we have

4 − 3(x + 2) + 2(3 − 2x)

4 − 3x − 6 + 6 − 4x

4 − 7x

Thus, the correct option is A.

More about the equivalent link is given below.

https://brainly.com/question/889935

#SPJ1

The missing options are given below.

4 − 7x

4 + 7x

7 − 4x

7 + 4x

This is one of my favorite probability problems. It uses many useful and powerful facts from probability.) Let X(t) be a stationary Gaussian random process with mX​(t)=0 and RX​(τ)=2e−5∣τ∣. Let Z=X(2)+ X(3). Find fZ​(z), the probability density function of Z

Answers

The probability density function of Z is fZ(z) = (1/√(2π(4 + 2e^(-5)))) * e^(-z^2/(2(4 + 2e^(-5))))

Given that X(t) is a stationary Gaussian random process with mX(t) = 0 and RX(τ) = 2e^(-5|τ|).

We are interested in finding the probability density function (PDF) of Z = X(2) + X(3).

First, we need to find the mean and variance of Z:

E[Z] = E[X(2) + X(3)] = E[X(2)] + E[X(3)] = 0 + 0 = 0

Var(Z) = Var(X(2) + X(3)) = Var(X(2)) + Var(X(3)) + 2Cov(X(2), X(3))

Since X(t) is a stationary process, we have:

Var(X(2)) = Var(X(3)) = RX(0) = 2

Cov(X(2), X(3)) = RX(1) = 2e^(-5)

Therefore, Var(Z) = 2 + 2 + 2e^(-5) = 4 + 2e^(-5)

Now we can use the properties of Gaussian random variables to find the PDF of Z. Since Z is a linear combination of Gaussian random variables, it is also Gaussian with mean 0 and variance 4 + 2e^(-5).

Thus, fZ(z) = (1/√(2π(4 + 2e^(-5)))) * e^(-z^2/(2(4 + 2e^(-5)))).

Therefore, the probability density function of Z is fZ(z) = (1/√(2π(4 + 2e^(-5)))) * e^(-z^2/(2(4 + 2e^(-5))))

To learn more about properties visit:

https://brainly.com/question/30034780

#SPJ11

employees at an antique store are hired at a wage of $15 per hour, and they get a $0.75 raise each year. write an equation that shows how a worker's hourly wage, y, depends on the number of years he or she has worked at the store,

Answers

To represent the hourly wage of an employee at the antique store, we can use the following equation:
y = 15 + 0.75x
where y represents the worker's hourly wage, and x represents the number of years the employee has worked at the store. In this equation, 15 is the initial hourly wage, and 0.75 is the annual raise.

The equation that shows how a worker's hourly wage, y, depends on the number of years he or she has worked at the store can be written as:

y = 15 + 0.75x

where x represents the number of years the employee has worked at the antique store.

This equation takes into account the starting wage of $15 per hour and the $0.75 raise that the employee receives each year they work at the store.

So, for example, if an employee has worked at the store for 5 years, their hourly wage would be:

y = 15 + 0.75(5) = 18.75

where y represents the worker's hourly wage, and x represents the number of years the employee has worked at the store.

Learn more about Number:

brainly.com/question/17429689

#SPJ11

A 12-sided dice has equal-sized faces numbered 1 to 12

a. Find P(number greater than 10)
b. Find P(number less than 5)
c. if the 12-sided dice is rolled 200 times, how many times would you expect either a 4,6, or 9 to be rolled

Answers

The solution is, the probability that exactly two of the dice show an even number is 0.3125.

We will use the Binomial Probability formula to find the answer

The formula is given by ⁿCₓ (p)ˣ (1-p)ⁿ⁻ˣ

Where:

n, the number of trials = 5

x, the number of success that we aim = 2

p, the probability of success = 0.5 ⇒ there are six out of twelve numbers on the dice that are even

Substitute these values into the formula, we have

P(2) = ⁵C₂ (0.5)² (1-0.5)⁵⁻²

P(2) = ⁵C₂ (0.5)² (0.5)³

P(2) = 0.3125

The solution is, the probability that exactly two of the dice show an even number is 0.3125.

To learn more on probability click:

brainly.com/question/11234923

#SPJ1

complete question:

Ben rolls 5 fair 12-sided dice. the 12 faces of each die are numbered from 1 to 12. what is the probability that exactly two of the dice show an even number?

TASK 5
What is the mean of the distribution below?
CODE: Enter the mean as your code. Do NOT round your answer. Enter all
decimal places. Put the decimal in the correct spot as well
10
END OF THE YEAR - ESCAPE ROOM
Movie Theme
X
X
Weights of Dogs
X
X
X
X
XX X X
15
20
X
X
X
X
25
Weight in pounds
X
X
30
Mtd
's M

Answers

The mean of the data-set in this problem is given as follows:

19.56 pounds.

How to calculate the mean of a data-set?

The mean of a data-set is given by the sum of all observations in the data-set divided by the number of observations, which is also called the cardinality of the data-set.

The dot plot shows the number of times that each observation appears, hence the observations are given as follows:

2 of 12 pounds.4 of 15 pounds.1 of 16 pounds.1 of 18 pounds.2 of 20 pounds.1 of 22 pounds.3 of 25 pounds.2 of 29 pounds.

Hence the mean is given as follows:

Mean = (2 x 12 + 4 x 15 + 1 x 16 + 1 x 18 + 2 x 20 + 1 x 22 + 3 x 25 + 2 x 29)/(2 + 4 + 1 + 1 + 2 + 1 + 3 + 2)

Mean = 313/16

Mean = 19.56 pounds.

More can be learned about the mean of a data-set at https://brainly.com/question/1136789

#SPJ1

You are studying a population of 1,800 wrestlers whose mean weight is 225 lbs with standard deviation of 20 lbs a) What proportion/percentage weight less than 220 lbs? b) What is the probability that a random wrestler weighs more than 250 lbs? c) How many wrestlers weigh between 210 and 230 lbs?

Answers

Approximately 670 wrestlers weigh between 210 and 230 lbs.

a) To find the proportion/percentage of wrestlers that weigh less than 220 lbs, we need to standardize the weight value using the formula:

z = (x - μ) / σ

where x is the weight value, μ is the mean weight, and σ is the standard deviation.

So, for x = 220 lbs:

z = (220 - 225) / 20 = -0.25

Looking up the standard normal table or using a calculator, we find that the area/proportion to the left of z = -0.25 is 0.4013. Therefore, the proportion/percentage of wrestlers that weigh less than 220 lbs is:

0.4013 or 40.13%

b) To find the probability that a random wrestler weighs more than 250 lbs, we again need to standardize the weight value:

z = (250 - 225) / 20 = 1.25

Using the standard normal table or a calculator, we find that the area/proportion to the right of z = 1.25 is 0.1056. Therefore, the probability that a random wrestler weighs more than 250 lbs is:

0.1056 or 10.56%

c) To find the number of wrestlers that weigh between 210 and 230 lbs, we first need to standardize these weight values:

z1 = (210 - 225) / 20 = -0.75

z2 = (230 - 225) / 20 = 0.25

Next, we need to find the area/proportion between these two standardized values:

P(-0.75 < z < 0.25) = P(z < 0.25) - P(z < -0.75)

Using the standard normal table or a calculator, we find that P(z < 0.25) is 0.5987 and P(z < -0.75) is 0.2266. Therefore:

P(-0.75 < z < 0.25) = 0.5987 - 0.2266 = 0.3721

Finally, we can find the number of wrestlers by multiplying this proportion by the total population size:

0.3721 * 1800 = 669.78 or approximately 670 wrestlers

Therefore, approximately 670 wrestlers weigh between 210 and 230 lbs.

To learn more about probability visit:

https://brainly.com/question/15124899

#SPJ11

1 point) A poll is taken in which 330 out of 550 randomly selected voters indicated their preference for a certain candidate. (a) Find a 90% confidence interval for p. 0.557 < p 0.642 (b) Find the margin of error for this 90% confidence interval for p. 0.042 (c) Without doing any calculations, indicate whether the margin of error is larger or smaller or the same for an 80% confidence interval. A. larger B. smaller C. same

Answers

This is because as the level of confidence decreases, the corresponding z-score becomes smaller, which in turn results in a smaller margin of error.

(a) To find a 90% confidence interval for the proportion p, we can use the formula:

CI = p ± z*(sqrt(p*(1-p)/n))

where p is the sample proportion, n is the sample size, and z is the z-score corresponding to the desired level of confidence (in this case, 90%).

We are given that p = 330/550 = 0.6 and n = 550. Using a standard normal distribution table, the z-score for a 90% confidence interval is approximately 1.645.

Substituting these values into the formula, we get:

CI = 0.6 ± 1.645*(sqrt(0.6*(1-0.6)/550))

= 0.6 ± 0.042

= (0.558, 0.642)

Therefore, a 90% confidence interval for the proportion of voters who indicated their preference for the candidate is 0.558 to 0.642.

(b) The margin of error for this 90% confidence interval is given by:

ME = z*(sqrt(p*(1-p)/n))

where z is the z-score corresponding to the desired level of confidence and p and n are as before.

Substituting the values we obtained earlier, we get:

ME = 1.645*(sqrt(0.6*(1-0.6)/550))

= 0.042

Therefore, the margin of error for this 90% confidence interval is 0.042.

(c) Without doing any calculations, we can see that the margin of error for an 80% confidence interval will be smaller than that for a 90% confidence interval. This is because as the level of confidence decreases, the corresponding z-score becomes smaller, which in turn results in a smaller margin of error.

To learn more about  visit:

https://brainly.com/question/1597341

#SPJ11

A spinner with repeated colors numbered from 1 to 8 is shown. Sections 1 and 8 are purple. Sections 2 and 3 are yellow. Sections 4, 5, and 6 are blue. Section 7 is red.

Spinner divided evenly into eight sections with three colored blue, one red, two purple, and two yellow.

Determine the theoretical probability of the spinner not landing on blue, P(not blue).

0.375
0.625
0.750
0.875

Answers

The theoretical probability of the spinner not landing on blue is 3/4

Determining the theoretical probability of the spinner not landing on blue

From the question, we have the following parameters that can be used in our computation:

Colors = 4 i.e. purple, yellow, blue and red

Blue = 1

Not blue = 3

So, we have

Theoretical probability = Not Blue /Colors

Substitute the known values in the above equation, so, we have the following representation

Theoretical probability = 3/4

Hence, theoretical probability is 3/4

Read more about probability at

brainly.com/question/251701

#SPJ1

Find the Maclaurin series for using the definition of a Maclaurin series. [Assume that has a power series expansion. Do not show that Rn(x) tends to 0.] Also find the associated radius of convergence. f(x)=/(1-x)^-2

Answers

Answer: i do not know but watch this: Find the Maclaurin series for f(x) = (1-x)^(-1) and associated radius of convergence by Ms. Shaws Math Class

Prove/disprove that the units/ones digit of 5221 is 3.

Answers

The units/one's digit of 5^221 is 5.

To determine the units/one unit digit of a number raised to a power, we only need to consider the units/one's digit of the base. In this case, the unit digit of 5 is 5.

Now, we need to look for a pattern in the units digit of 5 raised to different powers.

5^1 = 5 (units digit is 5)
5^2 = 25 (units digit is 5)
5^3 = 125 (units digit is 5)
5^4 = 625 (units digit is 5)
5^5 = 3125 (units digit is 5)
. . .and so on.

We can see that the unit digit of 5 raised to any power is always 5. Therefore, the units/one's digit of 5^221 is 5, not 3.

So, the statement "the units/one's digit of 5^221 is 3" is disproved.

Learn more about units:

https://brainly.com/question/141163

#SPJ11

The equation x^2+y^2-4x-8y-16=0 represents a circle in the standard xy-coordinate plane. What is the radius of the circle?

Answers

The radius of the circle represented by the equation x² + y² - 4x - 8y - 16 = 0 is given as follows:

6 units.

What is the equation of a circle?

The equation of a circle of center [tex](x_0, y_0)[/tex] and radius r is given by:

[tex](x - x_0)^2 + (y - y_0)^2 = r^2[/tex]

The equation for this problem is given as follows:

x² + y² - 4x - 8y - 16 = 0.

To obtain the radius of the circle, we must complete the squares from the equation, as follows:

x² - 4x + y² - 8y = 16

(x - 2)² + (y - 4)² = 16 + 2² + 4²

(x - 2)² + (y - 4)² = 36.

Hence the center and the radius of the circle are given as follows:

Center at (2,4).Radius of 6 units.

More can be learned about the radius of a circle at https://brainly.com/question/24375372

#SPJ1

I WILL GIVE BRAINLILEST TO WHOEVER GETS IT RIGHT

A 35-year-old person who wants to retire at age 65 starts a yearly retirement contribution in the amount of $5,000. The retirement account is forecasted to average a 6.5% annual rate of return, yielding a total balance of $431,874.32 at retirement age.

If this person had started with the same yearly contribution at age 40, what would be the difference in the account balances?

A spreadsheet was used to calculate the correct answer. Your answer may vary slightly depending on the technology used.

$378,325.90
$359,978.25
$173,435.93
$137,435.93

Answers

Answer:

B

Step-by-step explanation:

Using the same yearly contribution of $5,000 and an average annual rate of return of 6.5%, starting at age 40 instead of 35, the total balance at retirement age of 65 would be $359,978.25.

To calculate this, we can use the future value formula:

FV = PV x (1 + r)^n

where FV is the future value, PV is the present value (initial contribution), r is the interest rate per period, and n is the number of periods.

If we start at age 40 and contribute $5,000 per year for 25 years (until age 65), the present value would be $0 (since we haven't made any contributions yet) and the number of periods would be 25. The interest rate per period would be 6.5% / 1 = 0.065.

Using these values in the future value formula, we get:

FV = $5,000 x ((1 + 0.065)^25 - 1) / 0.065 = $359,978.25

Therefore, the difference in the account balances between starting at age 35 and starting at age 40 would be:

$431,874.32 - $359,978.25 = $71,896.07

So the correct answer is option B: $359,978.25.

From a lot of 14 missiles, 4 are selected at random and fired. Suppose the lot contains 3 defective missiles that will not fire. (a) What is the probability that all 4 missiles will fire? (b) What is the probability that at most 2 will not fire? (a) The probability that all 4 missiles will fire is ______ (Round to four decimal places as needed. ) (b) The probability that at most 2 will not fire is ______ (Round to four decimal places as needed. )

Answers

(a) The probability that all 4 missiles will fire is 0.2098 and (b) The probability that at most 2 will not fire is 0.0099.

Here we need to use the concept of combinations to get our required answer.

Here we have been given that 4 out of 14 missiles are defective.

Hence 10 missiles are working.

A sample of 4 missiles was chosen

Hence the simple can be chosen in ¹⁴C₄ ways

a)

The probability that all 4 missiles will fire is

¹⁰C₄/¹⁴C₄

= 0.2098

b)

The probability that at most 2 will not fire is

1 - the probability of including all three defective missiles

= 10³C₃/¹⁴C₄

= 0.0099

To learn more about Probability visit

https://brainly.com/question/30034780

#SPJ4

(a) Probability all 4 fire: 0.3297. (b) Probability at most 2 not fire: 0.9591.

(a) To track down the chance that all of the 4 rockets will fire, we actually need to do not forget the quantity of methods we can select 4 operating rockets out of the eleven that are not unsuitable, isolated by the all out range of approaches we will select four rockets out of the whole component.

The amount of ways of selecting four operating rockets out of the eleven that aren't improper is given by way of the combination recipe:

C(eleven,four) = 330

The absolute quantity of methods of selecting four rockets out of the entire part is given via:

C(14,4) = 1001

Subsequently, the chance that each one of the four rockets will hearth is:

P(all four hearth) = C(11,four)/C(14,4) = 330/1001 ≈ zero.3297

(adjusted to 4 decimal spots)

(b) To find the likelihood that at most 2 rockets might not fire, we need to remember every one of the potential conditions in which 0, 1, or 2 poor rockets are selected, and afterward song down the likelihood of every case and upload them up.

Case 1: No defective rockets are selected

The amount of ways of selecting four working rockets out of the eleven that are not faulty is given through the mixture recipe:

C(11,four) = 330

Subsequently, the chance of choosing no defective rockets is:

P(0 inadequate) = C(eleven,4)/C(14,four) ≈ 0.3297

Case 2: One defective rocket is chosen

The quantity of ways of selecting three operating rockets out of the eleven that are not poor is given with the aid of the combo equation:

C(eleven,three) = 165

The amount of ways of selecting 1 damaged rocket out of the three that are available is given through the mix equation:

C(three,1) = 3

Subsequently, the all out number of approaches of selecting 1 insufficient and three running rockets is:

C(eleven,three) × C(3,1) = 495

Thusly, the probability of choosing one deficient rocket and three running rockets is:

P(1 deficient) = C(eleven,three) × C(three,1)/C(14,4) ≈ 0.4655

Case three: Two poor rockets are selected

The amount of approaches of choosing 2 poor rockets out of the three which might be handy is given through the combination recipe:

C(3,2) = 3

The amount of ways of choosing 2 running rockets out of the 11 that aren't faulty is given via the mix recipe:

C(11,2) = 55

In this way, the all out number of methods of selecting 2 broken rockets and 2 working rockets is:

C(3,2) × C(eleven,2) = one hundred sixty five

Subsequently, the chance of choosing two incorrect rockets and two operating rockets is:

P(2 insufficient) = C(3,2) × C(11,2)/C(14,four) ≈ 0.1638

Subsequently, the likelihood that at maximum 2 rockets might not fireplace is:

P(at maximum 2 don't fire) = P(zero imperfect) + P(1 blemished) + P(2 deficient) ≈ 0.9591

(adjusted to 4 decimal spots).

To learn more about probability, refer:

https://brainly.com/question/1581511

#SPJ4

5.4 Diagonalization: Problem 6 (1 point) Suppose C=[1 2, 3 7], D=[2 0 , 0 1]
If A = CDC-1, use diagonalization to compute A5.
[ ]

Answers

To diagonalize C, we first need to find its eigenvalues and eigenvectors. The characteristic equation for C is det(C -

                                                                                                                     

                                λI)  =  0, which gives us (1 - λ)(7 - λ)  -  6  =                                

                                                                                                                     

                                   0. Solving for λ, we get λ1  =  1 and λ2  =                                    

                                                                                                                     

               7. To find the eigenvector corresponding to λ1, we solve the system of equations (C -                

                                                                                                                     

                   λ1I)x  =  0, which gives us the equation  - x1  +  2x2  =  0. Choosing x2  =                    

                                                                                                                     

                                           1, we get the eigenvector v1  =                                          

                                                                                                                     

                            [2,1]. Similarly, for λ2 we get the eigenvector v2  =  [1, -                            

                                                                                                                     

                              1]. We can then diagonalize C by forming the matrix P  =                              

                                                                                                                     

                         [v1, v2] and the diagonal matrix D  =  [λ1 0; 0 λ2]. We have C  =                        

                                                                                                                     

                      -                                    -                                                        

                   PDP 1. To compute A5, we first compute C 1 as [7  - 2;  - 3 1] / 4. Then, A  =                    

                                                                                                                     

                         -         -   -                                  5       5       5                          

                      CDC 1  =  PDP 1DC 1P. We have D  =  [1 0; 0 7], so D   =  [1  0; 0 7 ]  =                      

                                                                                                                     

                                           5       5 -                                                              

                    [1 0; 0 16807]. Thus, A   =  PD P 1  =  [2 1; 1  - 1][1 0; 0 16807][1 / 3  -                    

                                                                                                                     

                              1 / 3; 1 / 3 2 / 3]  =  [11203 11202; 16804 16805] / 9.                                

For more questions on diagonalisation - https://brainly.com/question/29537939

#SPJ11

The number of minutes Vinny spends playing her computer games in inversely proportional with her Math Models grade. If she spends 8 hours a week playing computer games, she has a 64. If she reduced her game playing time to 6 hours a week, what would her grade be in math models?

Answers

Vinny's new grade in math, if the playing time reduces to 6 hours per week, would be 85.33.

How to find the grade in math ?

If we wish to illustrate how Vinny's Math Models grade relates to the amount of time she spends playing computer games, we may employ the formula that defines an inverse proportion:

k = G x T

k = 64 x 8 = 512

For 6 hours playing:

512 = G x 6

G = 512 / 6

= 85. 33

Find out more on proportionality at https://brainly.com/question/29465808

#SPJ1

Can i get this by midnight or tmr thanks!

Answers

Answer: 1: C=72.3822947387, A=416.560184761, 2: C=37.6991118431, A=113.097335529

Step-by-step explanation:

For the first circle, since the area of a circle is equal to pi multiplied by the radius squared, we can use this equation:

[tex]\pi11.52^{2}[/tex]

Now we can multiply the radius by 2 to get the diameter, which is 23.04, and multiply the diameter by pi to get the circumference, 72.3822947387.

If you want an exact circumference, all you need to do is display the diameter multiplied by pi without simplification, which would look like this:

23.04π.

We can do the same thing for the second circle. Because we already have the diameter, I will start with the circumference.

12pi = 37.6991118431, which is just simplified to 12pi. The reason you want to have pi in your answer for an exact answer, is because otherwise your answer would have to be an irrational decimal constant, which cannot fit on one page.

Now for the area of the second circle. Divide the diameter by 2, and multiply pi by the radius squared. This is an equation that would look something like this:

[tex]\pi6^{2}[/tex], which can simplify to [tex]\pi36[/tex] as an exact answer.

Other Questions
You are a federal prosecutor attempting to use the Racketeering Influenced and Corrupt Organization (RICO) Act against an Asian organized crime group THAT operates a legitimate international clothing manufacturer in Brooklyn, New York. What provision of the RICO statutes are you going to use? mary read 1/7 pages on monday.there are 490 pages. how many pages did she read? Which of the taxes listed on her Form W-2 do you think Susan might be able to "getback?" companies that pay extra attention to the impact of sensations on product experiences are engaged in: question 73 options: exposure strategies sensory marketing tactile selling reception advertising Graph with x axis labeled Quantity Demanded and numbered in hundreds, 100 to 500. Y axis is Price, with prices 0 to 6. A line representing the Demand Curve starts at 100, 5 and drops at an angle down to 500, 1. Solid lines extend upward from the x axis to the green line at points 100, 5, 200, 4, 300, 3, 400, 2, and 500, 1. Public Domain Based on the graph, how is quantity demanded related to price? (5 points) Quantity demanded decreases as price decreases. Quantity demanded is equal to the product's price. Quantity demanded decreases as price increases. Quantity demanded increases as price increases. which of the following is the least helpful test for the assessment of thyroid disease? a. total T4 b. TSH c. free T4 d. antihyroid antibodies write the acidic equilibrium equation for hbro. be sure to include the proper phases for all species within the reaction. The Medication Appropriateness Index is designed to:1. Assess patient adherence.2. Assess the complexity of a patient's medication regimen.3. Provide a standardized method for medication risk assessment.4. Serve as a tool for communicating with patients about medication risk. What was the first investigation conducted by congress Internal and external conflicts in England, France, and the German states were primarily caused by which of the following in the period 1450 to 1750?religious conflicts between Christians and Muslimsreligious conflicts between Catholics and Protestantspolitical conflicts over territory in the Americaspolitical conflicts over the rules of inheritance Help me pls. paying a lot of points Brief Exercise 17-13 (Algo) Recording pension expense (LO17-7)Major Medical reported a net loss-AOCI in last year's balance sheet. This year, the company revised its estimate of future salary levels causing its PBO estimate to decline by $14 million. Also, the $26 million actual return on plan assets fell short of the $27 million expected return.How does this gain and loss affect Major's income statement statement of comprehensive income, and balance sheet? (Enter your answers in millions.) Income statement: Net Income Comprehensive Income statement: Gain-OCI Loss OCI Balance sheet: Net pension liability decreases by Vodne by some produces student grade ventweging voor. The converted 2.800 Vis five month of period. Al minden ole remained at there was a yo. sold each as follows mick on the son or the data) The company es traditional (norton courting Income atorments forts bankor. Saran would to the prese contribution Margien nome statements for era for management Read the requirements Compute the following amounts that would be shown on these income statements: Requirement 1. Gross Profit Identify the formula, then compute the gross profit. 11 Gross profit Il Requirement 2. Contribution Margin Identify the formula, then compute the contribution margin. 11 Contribution margin = Requirement 3. Total expenses shown below the gross profit line = Total expenses below the gross profit line Requirement 4. Total expenses shown below the contribution margin line + = Total expenses below the contribution margin line Requirements. Dollar value ofending inventory under absorption costing The dotar value of ending inventory under absorption contings Requirement 6. Doitar value of ending inventory under variable conting The dollar value of ending inventory under variable costing is Requirement 7. Which income statement will have a higher operating income Which option describes a narrator?(1 point)the events of a storythe person telling the storythe main idea of a storythe location of the story All of the following compounds can react as acids. Without using a table of acidities, rank them in order of increasing acidity. Explain your ranking.(a) CH3CH2SO3H (b) CH3CH2OH (c) CH3CH2COOH(d) CH3CHClCOOH (e) ClCH2CH2COOH the act or an instant of taking one's own life voluntarily and intentionally Aaden wants to get a subscription to a library. There are two subscription options one of which charges a fixed 96 dollar annual fee and the other which charges 3 dollars per book he borrows. what does point Q represent in this context?a. A number of books and their cost where the subscription with the annual fee costs lessb. A number of books and their cost where the subscription the charges per book costs lessc. A number of books and their cost where both subscriptions cost the same d. A number of books and their cost that is not possible with either subscription How knows how do this help me please 21 It is essential that you comply with government ...... Using the partial fractions technique, the function f(x) = 68x+168/ x^2+2x-24 can be written as a sum of partial fractions